Find the measure of the missing angle.
156°
a =

Answers

Answer 1

Answer:

Below!

Step-by-step explanation:

A straight line = 180 degrees, so all you need to do to find angle a is to subtract 156 from 180!

180 - 156 = 24 degrees

Hope this helped!


Related Questions

find the interior angle sum for the following polygon.​

Answers

We know that the interior angle sum of a polygon is (n − 2) × 180°, where n is the number of sides.

Here, number of sides (n) = 12

So, interior angle sum = (n − 2) × 180°

=> interior angle sum = (12 - 2) × 180°

=> interior angle sum = 10 × 180°

=> interior angle sum = 1800°

So, the interior angle sum of this polygon is 1800°.

A ladder reaches 6 m up a wall with its foot 2·4 m from the wall. A person standing under the ladder at a point 80 cm from its foot would be able to touch the ladder at a height of _____ m from the ground.​

Answers

Hey there! I'm happy to help!

We see that the ladder reaches a height of 6m from the ground while being 2.4 m from the wall. The ratio of the height to the length from the wall is 6:2.4 which can actually just simplify to 2.5 because 6m to is 2.4*2.5.

This person is 80 cm from the ladder's base. We just saw that you can take the distance from the wall (in this case, our wall is the human as the human is touching the height) and multiply it by 2.5 to find the height of the ladder at that specific point. So let's do that.

80*2.5= 200

We are looking for meters though. Since there are 100 centimeters in a meter, this is just going to be 2 meters.

Have a wonderful day! :D

The fox population in a certain region has a continuous growth rate of 7 percent per year. It is estimated that the population in the year 2000 was 14300. (a) Find a function that models the population t years after 2000 (t=0 for 2000). Hint: Use an exponential function with base e. Your answer is P(t)

Answers

Answer:

P(t) = 14300e^0.07t

Step-by-step explanation:

Let :

Population as a function of years, t = P(t) ;

Growth rate, r = 7%

Estimated population on year 2000 = Initial population = 14300

The given scenario can be modeled using an exponential function as the change in population is based in a certain percentage increase per period.

P(t) = Initial population*e^rt

P(t) = 14300*e^(0.07t)

P(t) = 14300e^0.07t

Where, t = number of years after year 2000.

Write an equation
in slope y-intercept form A(2,6),m=0

Answers

Solve for b:

y = 0x + b

6 = 0 + b

b = 6

The answer is y = 0x + 6

Part C Next, find the length of BC place point F at (4,4) and draw BF and FC now you hav the right ∆BFC with BC as the hypotenuse find BC and FC ising the coordinates of B,C, and,F then use the Pythagorean theorem to find BC show your work need help ASAP giving thanks and points away I just need these answer fast ???

Answers

Answer:

BF = |4 – 1| = 3

FC = |4 – 0| = 4

Using the Pythagorean Theorem to find BC:

BC2  =  BF2 + FC2

BC2   =  32 + 42

BC2   =  9 + 16

BC  =  sqaure root 25

BC  =  5

Step-by-step explanation:

The length of BC is 5 unit.

What is Pythagoras Theorem?

If ABC is a triangle with AC as the hypotenuse and angle B with 90 degrees then we have:

|AC|^2 = |AB|^2 + |BC|^2    

where |AB| = length of line segment AB. (AB and BC are rest of the two sides of that triangle ABC, AC being the hypotenuse).

We need to find the length of BC place point F at (4,4) and draw BF and FC now you have the right ∆BFC with BC as the hypotenuse find BC and FC using the coordinates of B,C, and,F then use the Pythagorean theorem to find BC.

Using the Pythagorean Theorem to find BC:

BC^2  =  BF^2 + FC^2

BC^2   =  32 + 42

BC^2   =  9 + 16

BC  =  √25

BC  =  5

So, BF = |4 – 1| = 3

FC = |4 – 0| = 4

Learn more about Pythagoras theorem here:

https://brainly.com/question/12105522

#SPJ2

value of the pronumeral and describe the rule.
4,1,-2,p,-8,-11​

Answers

Answer:

p = -5

every element in the sequence is created by subtracting 3 from the pervious element.

Step-by-step explanation:

I am not sure you put every necessary information here.

but I'd the visible information is truly everything, then it's is trivial.

the difference between 4 and 1 is ... well, -3. meaning we subtract 3 from 4 to get 1.

we suspect this is the rule and keep trying.

1 -3 = -2

hey it works.

and -8 -3 = -11

hey, also correct.

and the difference between -2 and -8 is -6, and when we place another item in between (p), we cut that in half again to -3. so, it is all consistent.

therefore,

p = -2 -3 = -5

the rule is

an = an-1 - 3

Correct answer get brainliest and 5 star

Answers

Answer:

B

Step-by-step explanation:

(Slant height)^2= (Radius)^2+(Height)^2

(20)^2= (12)^2+(Height)^2

Height=16

Answer:

B. pls mark me at brainliest and like my answer.

Step-by-step explanation:

⩥ɓeŋʝ⩤

Warren has money to go see a movie after he pays $8 for the MOVIE he will have $14 left right and solve an equation to show how much money Warren has before paying for the movie

Answers

Answer:

$(8+14)

Step-by-step explanation:

Amount of money Warren has at first=$8+$14=$22

If s = 6 and t = 4, find the value of x.
x = 4 + s - t​

Answers

Answer:

x = 6

Step-by-step explanation:

s = 6

t = 4

x = 4 + s - t

Substituting s and t in equation,

x = 4 + 6 - 4

x = 6

Answer:

6

Step-by-step explanation:

s=6

t=4

x= 4+6-4

x=10-4

x=6

Therefore; the final result is 6

What is the length of AC?

Answers

Answer:

B) 4 miles

Step-by-step explanation:

the big triangle CAB and small triangle CED are simular (given)

this means the ratio between their corresponding sides will be the same.

find corresponding sides with alternate interior angle theorem

m CDE= m CBA

vertical angles theorem

m ACB = m DCE

therefore, AC corresponds to CE

find ratio of sides

7 miles = 36960 feet

36960/ corresponding side 21 ft

36960/ 21= 1780 ft

multiply by 1780 to get from small side to big side

small side CE = 12 ft

12*1780

=21120 ft =AC (in ft)

convert to miles (ft to miles is ft/5280)

21120/5280

=4 miles

hope this helps

The perimeter of a rectangle is 74 inches. If the length is five more than the width, what are the rectangle's measurements?

Answers

Answer:

P=2(74)+2(5)

148+10=158

Answer my question

im being timed

Answers

Answer:

the third one

Step-by-step explanation:

what is |x-4| if x>4

Answers

Answer:

x-4

Step-by-step explanation:

If x>4, |x-4|>0 which means |x-4|=x-4

X = [?]°
Dr.
X Х
As
140°
B
C

Answers

Answer:

40°

Step-by-step explanation:

x+140=180°(angle on a straight line)

x=180-140

x=40°

∠DBA and ∠DBC are making linear pair.

Linear pair of angles are formed when two lines intersect each other at a single point.

The sum of angles of a linear pair is always equal to 180°

[tex] \rm \large\longrightarrow \: \: x \: + \: 140 \degree \: = \: 180 \degree[/tex]

[tex]\rm \large\longrightarrow \: \: x \: = \: 180 \degree \: - \: 140 \degree \: [/tex]

[tex]\rm \large\longrightarrow \: \: x \: = \: 40 \degree[/tex]

What is 15.4624 divided by 0.32

Answers

Answer:

48.32

Step-by-step explanation:

What is the value of x in the equation 1/5x - 2/3y
=
30, when y = 15?

Answers

Answer:

x = 200

Step-by-step explanation:

Plug in 15 as y into the equation and solve for x:

1/5x - 2/3y = 30

1/5x - 2/3(15) = 30

1/5x - 10 = 30

1/5x = 40

x = 200

So, when y = 15, x = 200

Find the Measure of one interior angle for each polygon

Answers

Answer:

5 corners : 108 degrees

6 corners : 120 degrees

Step-by-step explanation:

there are (at least) 2 different views to get the result :

officially (usually the teachers' preferred method) you consider a polygon as a combination of non-overlapping triangles. a polygon with n corners or edges we can split into n-2 such triangles.

each triangle has an angle sum of 180 degree.

so, the polygon angle sum is (n-2)×180 degrees.

and each (internal) angle is then (n-2)×180/n

n = 5 : (5-2)×180/5 = 3×36 = 108 degrees

n = 6 : (6-2)×180/6 = 4×30 = 120 degrees

the second approach (I prefer) goes after the external angles of the polygon.

the sum of all external angles in any polygon is 360 degrees (a full circle).

for n corners/edges each external angle is 360/n.

and the internal angle is then the complement to 180 degrees = 180 - 360/n

n = 5 : 180 - 360/5 = 180 - 72 = 108 degrees

n = 6 : 180 - 360/6 = 180 - 60 = 120 degrees

help pls pls pls pls pls pls

Answers

Answer Q=(9,-7). R=(9,-1). S=(5,-4)

Find the number of all the 2-digit numbers satisfying the
following congruences x = 3(mod7), x = 2(mod5).
OLEASE HELP

Answers

Use the Chinese remainder theorem.

Start with x = 3×5 + 2×7 = 15 + 14 = 29. Now,

• 29 ≡ 15 ≡ 1 (mod7)

• 29 ≡ 14 ≡ 4 (mod5)

Adjust for this by multiplying the first term in x by 3, and the second term by 3 (because 4×3 ≡ 12 ≡ 2 (mod5)).

So now x = 3×5×3 + 2×7×3 = 45 + 42 = 87, and

• 87 ≡ 45 ≡ 3 (mod7)

• 87 ≡ 42 ≡ 2 (mod5)

The CRT then says that x ≡ 87 (mod(7×5)) ≡ 87 (mod35), which is to say any number x = 87 + 35n satisfies both congruences (where n is any integer).

So there are 3 possible 2-digit numbers that work: {17, 52, 87}.

To confirm:

• 17 ≡ 15 + 2 ≡ 2 (mod5) and 17 ≡ 14 + 3 ≡ 3 (mod7)

• 52 ≡ 50 + 2 ≡ 2 (mod5) and 52 ≡ 49 + 3 ≡ 3 (mod7)

• 87 ≡ 85 + 2 ≡ 2 (mod5) and 87 ≡ 84 + 3 ≡ 3 (mod7)

Find the measure of b.
please help!

Answers

Answer:  40

=======================================================

Explanation:

The inscribed angle 20 degrees doubles to 2*20 = 40 which is the measure of the central angle, and the arc in which the inscribed angle subtends (or cuts off). This is due to the aptly named inscribed angle theorem.

------------

A slightly longer alternative path would be to do this:

The triangle with interior angles 20 and c is isosceles. Note how the missing angle up top is one of the congruent base angles, so the missing angle is 20 degrees. That means angle c is...

20+20+c = 180

40+c = 180

c = 180-40

c = 140

Then angle b is supplementary to this

b+c = 180

b+140 = 180

b = 180-140

b = 40

This path leads to the same answer. It's slightly longer, but it's a path you can take if you aren't familiar with the inscribed angle theorem.

In fact, this line of thinking is effectively how the inscribed angle theorem is proved as shown in the diagram below.

Laura brought some fish; salt water ($2 each) and fresh water ($1 each). If she has 15 fish - how many of each did she buy?

Answers

Answer:

are you sure there is no further info

Step-by-step explanation:

I need help right now!!!ASAP

Answers

Answer:

Points Q, J and M are not collinear

Step-by-step explanation:

Three points cannot be collinear if they are not coplanar (on the same plane), and M is on a different plane than Q. J is located on both planes.

pls explain me i will make u as brainlist​

Answers

Answer:

hope this help you

have a great day

an obtuse angle is an angle which is _______

Answers

Answer:

an obtuse angle is an angle that is more than 90 degrees

Step-by-step explanation:

A point is chosen randomly on KN. Identify the probability that the point is on KL or MN.

Answers

Answer:

11/18

Step-by-step explanation:

KL + MN = 11

Total = 18

Probability of KL or MN = 11/18

HELP ASAP KHAN NEED HELP NOW ITS KHAN

Answers

Answer:

[tex]{ \bf{ \green{line \: B}}}[/tex]

It's a direct proportionality

Any help would be nice

Answers

Answer:

11. angles on a straight line add up to 180

so 180 -133 equal to b.b=47

12.Vertical opposite angles are equal.

therefore b=50

-2z² + 4z + 2z² = ?
Anyone know this? ​

Answers

Answer:

4

Step-by-step explanation:

Let's simplify step-by-step.

−2z2+4z+2z2

Combine Like Terms:

=−2z2+4z+2z2

=(−2z2+2z2)+(4z)

=4z

Answer:

4z

Hope this helps <3 Need thanks?

Comment /hearthelp

[tex]\\ \sf\longmapsto -2z^2+4z+2z^2[/tex]

Combine like. variables

[tex]\\ \sf\longmapsto -2z^2+2z^2+4z[/tex]

[tex]\\ \sf\longmapsto (-2+2)z^2+4z[/tex]

[tex]\\ \sf\longmapsto 0z^2+4z[/tex]

[tex]\\ \sf\longmapsto 4z[/tex]

Help me, please
I really need

Answers

Answer:

answrr

yes ok

Step-by-step explanation:

4+4-4#_44

You and six friends play on a basketball team. A sponsor paid $100 for the league fee, x dollars for each player’s T-shirt, and $68.25 for trophies. Write an expression for the total amount paid by the sponsor

Answers

Answer:

Total amount paid by the sponsor = 175 + 6d

Step-by-step explanation:

You and 5 friends = 6 people

Cost of renting a bus = $75

Team entry fee = $100

Cost of each student t shirts = $d

Cost of 6 student t shirts = $d × 6= $6d

Write an expression for the total amount the sponsor paid.

Total amount paid by the sponsor = Cost of renting a bus + Team entry fee + Cost of 6 student t shirts

= $75 + $100 + $6d

= $175 + $6d

Total amount paid by the sponsor = 175 + 6d

Where,

d = cost of each student t shirts

Other Questions
Immigrants into Spanish Texas who came from mixed Spanish and Native American heritage were called______________.a.Lipan Apachesb.Mestizosc.Vaquerosd.Canary Islanders Find an equation in slope-intercept form of the line that has slope 7 and passes through point A(8,-4)a.) y = 52x + 7b.) y = 7x + 52c.) y = -7x - 52d.) y = -7x + 52 Decide whether the graphs of the two equations are parallel. y=3x+8 and y/3-3=x write a letter to editor about garbage management in your locality? x^3 -2mx^2 +16 (x+2)PLS ANSWER FASTI WILL MARK YOU THE BRANLIEST What can descriptive details help a reader understand? Check all that apply.what something looks likewhat something smells likehow something soundshow a word is spelledwhat something feels like Portillo's Fast Food restaurants are changing their hot dog vendor. The company has realized that in order to stay competitive, its hot dogs need to be made larger and shorter so that they can fit in a new smaller-sized hot dog bun. Consequently, Portillo's is looking for a vendor that can provide a product according to its new specifications at a price that is less than what it was paying in the past. At what stage in the business buying process is Portillo's Draw a wave that has a wavelength of 3 cm and an amplitude of 1 cm. Label the wavelength, the amplitude, the rest position, and the crest and trough of your wave. What is the x-coordinate of the point of intersection for the two lines below?-6 + 8y = -67x -10y = 9 Answer choices 1.) -62.) -33.) 34.) 7 Select all correct answers What are the solution to this equation -7+(x^2-19)^3/4=20 Find the measure of a. Gii cc bt phng trnh v biu din tp nghim trn trc s(X-1)(x+2)>(x-1)(x-1) +3 Question 7 of 10What is the slope of the line described by the equation below?y-9 = -2(x-8) Butter kruse made 522 glaze donuts and 323 chocolate covered donuts. They put the donuts in boxes. Each box holds 12 donuts. How many boxes do they need? ei 7-1Which method would result in highest probability for any of the results?O rolling a dieO flipping a coinO spinning a wheel with 4 different colorsOpicking a red marble out of a jar of 10 different colored marblesANSWER PLEASEEEEE A flag has a perimeter of 5 metres. The length of the flag is 600 mm more than thewidth of the flag. The length is represented by L and the width is represented by W.Which of the linear systems in the choices represents the description?O2L + 2W = 5L - 600 = WO2L + 2W = 5000L - 600 = WOL+W = 5000W + 600 = LO L x W = 5000L + 600 = W 14 a and b please help (Choose the most appropriate answer from the choices below) In an assembly line balancing problem, work station cycle time.I. will decrease as the required production per day increases, other things remaining the same.II. will increase as the required production per day increases, other things remaining the same.III. will decrease as the production time per day increases, other things remaining the same.IV. will increase as the production time per day increases, other things remaining the same.A. IVB. IIC. ID. IIIE. I. and IV The supreme choice pizza at Paolo's Pizzeria contains 3 different meats and 5 different vegetables. The customer can select any one of 4 types of crust. If there are 5 meats and 7 vegetables to choose from, how many different supreme choice pizzas can be made? Complete the following statement using the choices below. Air moves out of the lungs when the pressure inside the lungs is ________. less than the pressure in the atmosphere greater than the intra-alveolar pressure greater than the pressure in the atmosphere equal to the pressure in the atmosphere